PT64.S3.Q26 - acquiring complete detailed information

NotMyNameNotMyName Alum Member Sage
edited October 2017 in Logical Reasoning 5320 karma

https://7sage.com/lsat_explanations/lsat-64-section-3-question-26/

I got this question right during the PT by POE (A-B reverses necessary and sufficient terms, ACD all use most-statements) but in BR I really had to labor over the logic. It in fact took several sessions to feel good about my interpretation. However, my translation differs slightly from JY due to the presence of nested terms so I'm curious what ya'll think about it.

expect benefits outweigh cost → (acquire info → R)

Therefore

acquire info → R

This appears to be the most basic logical structure in the book. A→B, therefore B. We need A. But the nested terms muddy the waters a bit. We need to conclude R which is nested within the necessary condition. Because of the odd form, there are more ways of doing that than just A→B. But, most importantly, we can't conclude R if one expects the "benefits outweigh the cost".

This is really where I had my difficult. the easiest correct AC would have just said "The benefits never outweigh the cost". However, what they gave us was

acquire infoexpect benefits outweigh cost

That results in a perfectly valid but unusual form. Our final chain looks like

expect benefits outweigh costacquire info → R

"Consumers who do not bother to acquire this info are acting rationally". We can safely say that now because consumers who don't bother to acquire this info also don't expect the benefits to outweigh the costs.

Comments

  • AllezAllez21AllezAllez21 Member Inactive Sage Inactive ⭐
    1917 karma

    You seem to have gone the diagram heavy route.

    My main difference in looking at this question is that it turns on the word "expect." We are talking about the beliefs of a group, not the actual reality of a situation. So I think even if the benefits never outweighed the cost, that wouldn't guarantee the conclusion, because we need something about expectations.

    Basically, from a non diagram perspective, we are given that an action is presumably rational in all cases except (unless) when expectations are that benefits outweigh costs. So to make the conclusion valid, we just need to do away with the one exception. E does this.

  • NotMyNameNotMyName Alum Member Sage
    5320 karma

    @AllezAllez21 Yea that makes sense. Under timed conditions, I didn't diagram at all... just eliminated A-D because they were clearly wrong.

    Basically, from a non diagram perspective, we are given that an action is presumably rational in all cases except (unless) when expectations are that benefits outweigh costs. So to make the conclusion valid, we just need to do away with the one exception. E does this.

    You've condensed my entire explanation into 2 very clear sentences. That's exactly it and way more obvious than diagramming everything. I like how you replaced "unless" with "except" that unlocked the entire picture for me.

Sign In or Register to comment.